In circle o, a diameter has endpoints (-5,4) and(3, -6). What is the length of the diameter?Answer choices: a.) sqrt 10b.) 2sqrt 41c.) sqrt 2 d.) 2sqrt 2

Answers

Answer 1

Diameter has endpoints (-5,4) and (3, -6).

the endpoints of the diameter = (-5,4) and (3,-6)

The length of diameter can be calculated by sqrt 10,11

Distance between two coordinates (-5,4) and (3,-6).

Distance formula is express as

[tex]\begin{gathered} \text{Distance = }\sqrt[]{(x_2-x_1)^2+(y_2-y_1)^1^{}} \\ \text{Distance}=\sqrt[]{(3-(-5))+(-6+(4))} \\ \text{Diamter = }\sqrt[]{8-(-2)} \\ \text{Diameter = }\sqrt[]{10} \end{gathered}[/tex]

Diameter = sqrt 10

Answer: Diameter = sqrt 10


Related Questions

A pyramid has a square base with sides of length s. The height of the pyramid is equal to of the length of a side on the base. Which formularepresents the volume of the pyramid?OA. V = ¹8³OB. V=¹8³OC. V=8³OD. V=35³OE V=65³

Answers

Given,

The measure of the length of the side of square is s.

The height of the pyramid is 1/2 of the length of the side.

Required

The volume of the square pyramid.

The formula for the volume of the square pyramid is,

[tex]Volume\text{ =}\frac{side\times side\times height}{3}[/tex]

Substituting the values then,

[tex]\begin{gathered} Volume\text{ =}\frac{s\times s\times s}{3\times2} \\ =\frac{s^3}{6} \end{gathered}[/tex]

Hence, the volume of the pyramid is s^3/6.

The lowest airport in the world is Atyrau Airport in Kazakhstan at 72 feet below sea level. How would we represent this elevation using negative numbers? 6.NS.5

Answers

ANSWER

-72 ft

EXPLANATION

We can draw a number line:

If the 0 of the number line is sea level, we would normally say that all positive numbers are those that are above sea level and negative numbers those that are below sea level.

Therefore, to represent an elevation that is below sea level, we'd write it as a negative number: -72 ft

Assume the random variable X has a binomial distribution with the given probability of obtaining a success. Find the following probability, given the number of trials andthe probability of obtaining a success. Round your answer to four decimal places.P(X ≤ 4), n = 8, p = 0.4

Answers

Answer: [tex]\text{P\lparen X \le4\rparen is 0.8263}[/tex]Explanation:

Given:

n = 8, p = 0.4

To find:

P(X ≤ 4)

To determine the given probability, we will apply the binomial probability formula:

[tex]\begin{gathered} $$P(x=X)=^nC_x\times p^x\times q^{n-x}$$ \\ n\text{ = number of trials} \\ p\text{ = number of success} \\ q\text{ = number of failure} \end{gathered}[/tex][tex]P\left(X≤4\right)\text{ = P\lparen x = 0\rparen + P\lparen x = 1\rparen + P\lparen x =2 \rparen + P\lparen x = 3\rparen + P\lparen x = 4\rparen}[/tex][tex]\begin{gathered} p\text{ + q = 1} \\ q\text{ = 1- p} \\ q\text{ = 1 - 0.4} \\ q\text{ = 0.6} \\ \\ P(x=0)=\text{ }^8C_0\times(0.4)^0\times(0.6)^{8-0} \\ P(x\text{ = 0\rparen = 0.01679616} \\ \\ P(x=1)=\text{ }^8C_1\times(0.4)^1\times(0.6)^{8-1} \\ P(x\text{ = 1\rparen = 0.08957952} \end{gathered}[/tex][tex]\begin{gathered} P(x=2)=\text{ }^8C_2\times(0.4)^2\times(0.6)^{8-2} \\ P(x\text{ = 2\rparen = 0.20901888} \\ \\ P(x=3)=\text{ }^8C_3\times(0.4)^3\times(0.6)^{8-3} \\ P(x\text{ = 3\rparen = 0.27869184} \\ \\ P(x=4)=\text{ }^8C_4\times(0.4)^4\times(0.6)^{8-4} \\ P(x\text{ = 4\rparen = 0.2322432} \end{gathered}[/tex][tex]\begin{gathered} P(X≤4)\text{ = 0.01679616 + 0.08957952 + 0.20901888 + 0.27869184 + 0.2322432} \\ \\ P\left(X≤4\right)\text{ = 0.8263296} \\ \\ To\text{ 4 decimal place, P\lparen X \le4\rparen is 0.8263} \end{gathered}[/tex]

What is the value of a + b+c? you may assume that the ray is tangent to the circle?a. 86b.150c.133d.47

Answers

ANSWER:

c. 133°

STEP-BY-STEP EXPLANATION:

We have that there is a chord that divides the circle in two equal parts because it passes through the middle of the circle. Since it is half of the circle, the arc is 180°, we can see that this arc is the sum of the angles a + b, this angle must measure half of 180°, therefore:

[tex]a\degree+b\degree=\frac{180\degree}{2}=90\degree[/tex]

Now the angle c° must measure half of the 86° arc, therefore:

[tex]\begin{gathered} c\degree=\frac{86\degree}{2}=43\degree \\ \end{gathered}[/tex]

That means that a° + b° + c° is equal to:

[tex]a°+b°+c°=90+43=133\degree[/tex]

Therefore, the correct answer is c. 133°

A county is planning to expand its train service. To better understand the current service, the county planner looked at which train stations are along or not along various train lines

Answers

Solution

(a)

5 stations are along the orange line

(b)

8 stations

Which of the following is equivalent to a rhombus? *O A rectangle with congruent diagonalsA parallelogram where opposite angles are congruentA parallelogram with two consecutive congruent sidesA quadrilateral where consecutive angles are supplementary

Answers

A parallelogram with two consecutive congruent sides.

1) Since a rhombus is a quadrilateral with 4 congruent sides. Then we can examine the options and state that

2) Given that a rhombus has congruent opposite angles, with four congruent sides.

3) The answer is

A parallelogram with two consecutive congruent sides.

With two consecutive sides, the whole polygon will have congruent sides.

C Campus StudentCampus StudentGA-051 st AFJROTCGA-051 st AFJROTC5 New Tabebra_TC_Online LearningSubtracting with a Model3Subtract: 95 - 43Click or tap blocks to subtract them.032O 42O 52O 62

Answers

95 - 43= 52

9 5

- 4 3

______

5 2

____________

5-3 =2

9-4 = 5

______________

Answer

The third choice, 52

find the surface area of a composite figure round to the nearest tenth if necessary to units

Answers

The composite image is that of a cuboid and a triangular prism

For the cuboid, the surface area will be

A = LB + 2BH + 2LH

L = 1.8

B= 1.1

H= 0.8

A = 1.8X1.1 + 2 X 1.1 X 0.8 + 2 X 1.8 X 0.8

A = 1.98 + 1.76 +2.88

Area of cuboid = 6.62

In 2010, the population of a city was 170,000. From 2010 to 2015, the population grew by 4.5%. From 2015 to 2020, it fell by 3.3%. To the nearest 100 people, what was the population in 2020?

Answers

Given a percentage of change we can calculate the final population by the end of a period with the following formula:

P = P0 + xP0

Where P is the final population, P0 is the initial population at the beginning of the period and x is the percentage of change, if the population grew x will be a positive value and if the population fell x will be a negative value.

For the period 2010 to 2015, the population was initially 170,000 and it grew by a 4.5%. Then by replacing 0.045 for x (4.5/100) and 170000 for P0, we get:

P2015 = 170000 + 0.045×170000 = 177650

From 2015 the population decay by 3.3%, then we can calculate the population by 2020 to get:

P2020 = 177650 + 177650×(-0.033) = 171787

Then, rounding to the nearest 100 people we get 171,800

units givenQuestion 2 (3 points)toUsing the Rectangular Prism in the picture, find the Lateral Area, the Area of a Single Base,and the TOTAL Surface Area:.12 cm825 cm10 cmUse the face with dimensions 10cm x 5cm as the base.Lateral Area =cm²Single Base Area =64188 in48cm²

Answers

Solution:

Given the rectangular prism;

Where;

[tex]\begin{gathered} length,l=10cm \\ \\ width,w=5cm \\ \\ height,h=2cm \end{gathered}[/tex]

Thus, the lateral area, L is;

[tex]\begin{gathered} L=2(l+w)h \\ \\ L=2(10+5)2 \\ \\ L=4(15) \\ \\ L=60cm^2 \end{gathered}[/tex]

ANSWER:

[tex]Lateral\text{ }Area=60cm^2[/tex]

Also, the single base area, B, is;

[tex]\begin{gathered} B=l\times w \\ \\ B=10\times5 \\ \\ B=50cm^2 \end{gathered}[/tex]

ANSWER:

[tex]Single\text{ }Base\text{ }Area=50cm^2[/tex]

Then, the surface area, S, is;

[tex]\begin{gathered} S=L+2B \\ \\ S=60+2(50) \\ \\ S=60+100 \\ \\ S=160cm^2 \\ \\ \end{gathered}[/tex]

ANSWER:

[tex]Surface\text{ }Area=160cm^2[/tex]

Garrick gets paid $4.70 an hour with time-and-a-half for overtime(over 40 hours). How much did he earn one week when he worked 46hours?a. $195.05b. $220.90c. $230.30d. $188

Answers

Solution

For this case we can calculate the total with the following formula:

Fixed amount= 40*4$.70= $188

And now for the remain hours we can do this:

Extra = 6* 1.5*$4.70= $42.3

Then the total amount is:

Fixed amount+ Extra = 188$ + 42.3$ = $230.3

Then the answer is:

c. $230.30

if there are 36 successful outcomes in a sample size of 80 what is the sample proportion

Answers

The sample proportion is the number of successes over the sample size. That is:

36/80 = 9/20

The number of fish in a fish tank doubles each week. The function y=equals 3 (2)^x represents the population, where X is the number of one week periodsa. Describe the domains and range of the function. Then graph the functionb. Find and interpret the y intercept c. How many fish are in the tank at the end of first week?d. How many fish are in the tank after 4 weeks?

Answers

a. The domain of the function is the set of values of the independent variable (x) on which the function acts, in this case, the independent variable is time, since the time can only take positive values, the domain would be [0,∞) or x≥0.

The range is all the possible values that the dependent variable can take, in this case, the dependent variable is the population of fish, then it can't be a negative number, since you can't have for example -1 fish, then, again, the interval would be [0,∞) or y≥0

A graph of the function looks like this:

b. We can find the y-intercept by making x equals 0 in the formula of the function, like this:

y= 3*(2)^x

y= 3*(2)^0

y= 3*1, since any number raised to zero equals 1

y=3

Since x equals zero represents the initial time, the y-intercept represents the initial population of fish, then at the beginning, there were 3 fish.

c. We just have to replace 1 for x, and then calculate y, like this:

y= 3*(2)^x

y= 3*(2)^1

y= 3*(2)

y=6

At the end of the first week, there will be 6 fish.

d. Now, we just have to replace 4 for x, like this:

y= 3*(2)^4

y= 3*16

y=48

Then, after 4 weeks, there will be 48 fish

he following list contains the average annual total returns (in percentage points) for 9 mutual funds. The mutual funds appear in an online brokerage firm'sall-star" list.-9, 23, 12, 4, 11, 5, 36, 7, 31Send data to calculator(a) What is the mean of this data set? If your answer is not aninteger, round your answer to one decimal place.(b) What is the median of this data set? If your answer is notan integer, round your answer to one decimal place.(c) How many modes does the data set have, and what aretheir values? Indicate the number of modes by clicking in theappropriate circle, and then indicate the value(s) of themode(s), if applicable.00zero modesone mode:two modes: andX

Answers

Given

average annual total returns for 9 mutual funds.

-9, 23, 12, 4, 11, 5, 36, 7, 31

Find

a) Mean

b) Median

c) Mode

Explanation

a) Mean = sum of observations/ total number of observation

so ,

[tex]\begin{gathered} mean=\frac{-9+23+12+4+11+5+36+7+31}{9} \\ \\ mean=\frac{120}{9} \\ \\ mean=13.33333\approx13.3 \end{gathered}[/tex]

to find median , we need to arrange in ascending order :

-9 , 4 , 5 , 7 , 11 , 12 , 23 , 31 , 37

there are 9 entries which is an odd number

so , median = (n+1/2)th term

[tex]\begin{gathered} \frac{9+1}{2} \\ \frac{10th}{2} \\ 5th \end{gathered}[/tex]

so , median = 11

there is no mode because no term is repeating.

Final Answer

Hence ,

mean = 13.3

median = 11

mode - zero mode

Complete the remainder of the table for the given rules

Answers

To complete the table you have to input each value of x in the given equation and solve for y:

Function:

[tex]y=-2x+9[/tex]

For x= -2

[tex]\begin{gathered} y=-2\cdot(-2)+9 \\ y=4+9 \\ y=13 \end{gathered}[/tex]

x= -2 → y=13

For x= 0

[tex]\begin{gathered} y=-2\cdot0+9 \\ y=0+9 \\ y=9 \end{gathered}[/tex]

x=0 → y=9

For x= 2

[tex]\begin{gathered} y=-2\cdot2+9 \\ y=-4+9 \\ y=5 \end{gathered}[/tex]

x=2 → y=5

For x= 4

[tex]\begin{gathered} y=-2\cdot4+9 \\ y=-8+9 \\ y=1 \end{gathered}[/tex]

x=4 → y=1

factorize 5m^3-10m^2+8m+2

Answers

The expression 5m³ - 10m² + 8m + 2 cannot be factorized

How to factorize the polynomial?

From the question, the polynomial is given as

5m^3-10m^2+8m+2

Rewrite the polynomial properly

So, we have

5m³ - 10m² + 8m + 2

Next, we represent the above expression on a graph

From the attached graph, we can see that the polynomial expression only cross the x-axis at x = -0.197

This implies that the expression cannot be factored

So, we cannot further rewrite the given expression

Read more about factorized expressions at

https://brainly.com/question/2750166

#SPJ1

The maximum value in this range is: Use the range rule of thumb to determine whether 1 girl in 10 births is a significantly low number of girls.

Answers

Answer:

The maximum value in the range is 8.113

1 girl in 10 births is a significantly low number of girls.

Explanation:

Note that the range rule of thumb says that the range of about 4 times the standard deviation.

We'll use the below formula to determine the standard deviation;

[tex]\begin{gathered} \sigma=\sqrt[]{\lbrack\sum^{}_{}x^2\cdot P(x)\rbrack-\mu^2} \\ \text{where }\mu=\text{ population mean} \end{gathered}[/tex]

Let's go ahead and determine the mean as seen below;

[tex]\mu=\sum ^{}_{}\lbrack x\cdot P(x)\rbrack[/tex][tex]\begin{gathered} \mu=(0\cdot0.005)+(1\cdot0.12)+(2\cdot0.039)+(3\cdot0.113)+(4\cdot0.196)+(5\cdot0.235)+(6\cdot0.209) \\ +(7\cdot0.113)+(8\cdot0.036)+(9\cdot0.016)+(10\cdot0.026) \end{gathered}[/tex][tex]\begin{gathered} \mu=0.12+0.078+0.339+0.784+1.175+1.254+0.791+0.288+0.144+0.26 \\ \mu=5.233 \end{gathered}[/tex]

Let's now determine the below;

[tex]\begin{gathered} \sum ^{}_{}x^2\cdot P(x)=(0^2\cdot0.005)+(1^2\cdot0.12)+(2^2\cdot0.039)+(3^2\cdot0.113)+(4^2\cdot0.196)+(5^2\cdot0.235) \\ +(6^2\cdot0.209)+(7^2\cdot0.113)+(8^2\cdot0.036)+(9^2\cdot0.016)+(10^2\cdot0.026) \end{gathered}[/tex][tex]\begin{gathered} \sum ^{}_{}x^2\cdot P(x)=0.012+0.156+1.017+3.136+5.875+7.524+5.537+2.304+1.296+2.6 \\ =29.457 \end{gathered}[/tex]

So the standard deviation will be;

[tex]\sigma=\sqrt[]{29.457-5.233^2}=\sqrt[]{29.457-27.384}=\sqrt[]{2.073}=1.44[/tex]

Let's determine the maximum and minimum value of the distribution as seen below;

[tex]\begin{gathered} Maximum\text{ value = }\mu+2\sigma=5.233+2(1.44)=5.233+2.88=8.113 \\ \text{Minimum value }=\mu-2\sigma=5.233-2(1.44)=5.233-2.88=2.353 \end{gathered}[/tex]

We can see from the above that the number of girls born among 10 children should be between the range of 2.353 and 8.113, therefore 1 girl in 10 births is a significantly low number of girls.

The maximum value in this range is 8.113

At store A, oranges are $3.99 for 5 apples. At store B oranges are $20 for 7 apples which is the better deal?

Answers

Answer:

Store A

Explanation:

To know which is the best deal, we need to find the price per apple for each store, so we need to divide the price by the number of apples.

For store A:

[tex]\frac{\text{ \$3.99}}{5\text{ Apples}}=0.798\text{ per apple}[/tex]

For Store B:

[tex]\frac{\text{ \$20}}{7\text{ Apples}}=2.85\text{ per apple}[/tex]

So, the best deal is Store A because the price per apple is less than the price of Store B.

Hi how are you today can you please help me with this question

Answers

$ 119.607

Explanation

we can easily solve this by using a rule of three

Step 1

Let

actual price(2020)=price of 2019 + 2%

so

actual price(2020)= 102 % and

price ( 2019)=100 %

let x represents the cost for 2019, so

[tex]\begin{gathered} \text{if} \\ 122\Rightarrow\text{ 102\%} \\ x\text{ }\Rightarrow\text{ 100 \%} \end{gathered}[/tex]

the proportion would be

[tex]\begin{gathered} \frac{122}{102\text{ \%}}=\frac{x}{100\text{ \% }} \\ \text{ Multiply both sides by 100 \%} \\ \frac{122}{102\text{ \%}}\cdot100=\frac{x}{100\text{ \% }}\cdot100 \\ 119.607=x \end{gathered}[/tex]

therefore, the cost in January of 2019 is

$ 119.607

I hope this helps you

2500/10.5 please show work

Answers

The given expression is

2500/10.5

We would multiply the numerator and denominator by 10. We have

2500 x 10/10.5 x 10

= 25000/105

We would divide the numerator and denominator by common factors until they can't be simplified further. Dividing by 5, we have

5000/21

It can't be simplified further since there is no common factor of 5000 and 21. We would convert the fraction to mixed fraction

5000/21 = 238 remainder 2

Thus, if we write it as mixed fraction, we have

238 2/21

Find the circumference of this circleusing 3 for T.C ~ [?]14C = 27r

Answers

Hello! To calculate the circumference, we have to use the formula below:

[tex]C=2\cdot\pi\cdot r[/tex]

pi = 3

radius (r) = 14

Replacing these values in the formula, we will have:

[tex]\begin{gathered} C=2\cdot3\cdot14 \\ C=6\cdot14 \\ C=84 \end{gathered}[/tex]

A survey was conducted to determine the food choices of the 80 students at a picnic. The types of food are in the graph belowSalad 10%Sandwich 20%Hamburger 15%Hotdog 15%Pizza 30%Based on the graph how many more students chose pizza than students who chose salad

Answers

First, find the amount of students who chose pizza and the amount of students who chose salad based on the total amount of students in the survey and the corresponding percentages.

Pizza: 30%

[tex]\frac{30}{100}\times80=24[/tex]

Then, 24 students chose pizza.

Salad: 10%

[tex]\frac{10}{100}\times80=8[/tex]

Then, 8 students chose salad.

Subtract the amount of students who chose salad (8) from the amount of students who chose pizza (24) to find how many more students chose pizza than students who chose salad.

[tex]24-8=16[/tex]

Therefore, there are 16 more students who chose pizza than students who chose salad.

What is the volume of this cube? 2 cm 2cm 2cm

Answers

Answer:

The volume is

[tex]8\operatorname{cm}^3[/tex]

Explanation:

The volume of a cube with length l is given by the formula:

[tex]V=l^3[/tex]

Given the length 2 cm

The volume is:

[tex]V=2^3=8\operatorname{cm}^3[/tex]

Find the largest angle of △TUV. Assume that s is a positive number.

Answers

Remember that

In any triangle: The shortest side is always opposite the smallest interior angle. The longest side is always opposite the largest interior angle

so

In this problem

the largest interior angle is opposite to the longest side (TU)

that means

the largest interior angle is

I need help with this question please1) Picture2) In(2x) + ln(7) = 4

Answers

1) We must solve for x the following equation:

[tex]e^{5x}=25.[/tex]

To solve this equation, we take the natural logarithm to both sides of the equation:

[tex]\begin{gathered} \ln (e^{5x})=\ln (25), \\ 5x\cdot\ln e=\ln 25. \end{gathered}[/tex]

Now, we use the following results:

[tex]\begin{gathered} \ln e=1, \\ \ln (25)=\ln (5^2)=2\cdot\ln 5. \end{gathered}[/tex]

Replacing these results in the equation above, we have:

[tex]5x=2\cdot\ln 5.[/tex]

Solving for x, we get:

[tex]x=\frac{2}{5}\cdot\ln 5\cong0.64.[/tex]

2) We must solve for x the following equation:

[tex]\ln (2x)+\ln (7)=4.[/tex]

To solve this problem, we isolate the part that involves the x:

[tex]\ln (2x)=4-\ln (7)\text{.}[/tex]

Now, using the following property:

[tex]\ln y=z\rightarrow y=e^z\text{.}[/tex]

with:

[tex]\begin{gathered} y=2x, \\ z=4-\ln 7. \end{gathered}[/tex]

we have:

[tex]\ln (2x)=4-\ln 7\rightarrow2x=e^{4-\ln 7}.[/tex]

Solving the last equation for x, we get:

[tex]x=\frac{1}{2}\cdot e^{4-\ln 7}\cong3.90.[/tex]

Answers

1) The value of x that solves the first equation is 0.64 to two decimal places.

2) The value of x that solves the second equation is 3.90 to two decimal places.

Review of the base of a logarithm

We can define the logarithm in base a through the following equations:

[tex]\begin{gathered} \log _aa=1, \\ \log _aa^x=x\cdot\log _aa=x\cdot1=x\text{.} \end{gathered}[/tex]

When we use as a base the Euler number e ≅ 2.718, the logarithm is called "natural" and we use the following notation for it:

[tex]_{}\log _e=\ln .[/tex]

With this notation, we have the following properties:

[tex]\begin{gathered} \ln e=1, \\ \ln e^x=x\cdot\ln e=x\cdot1=x\text{.} \end{gathered}[/tex]

Shaun estimated that the attendance at a college basketball game was 4,000. The actual attendance was 3,475. What is the percent error of Shaun's estimate? Round to the nearest whole percent.

Answers

Percentage = 100 * (4000 - 3475)/3475 = 100 * (525/3475) = 52500/3475 = 15.1 %

Answer:

15.1%

Ronald was 1.5 times olderthan Megan. If Ronald was 27years old, how old is Megan?Write an equation to solve.

Answers

Solution:

Let x represent Ronald's age, and y represent Megan's age.

Thus,

[tex]\begin{gathered} x\Rightarrow Ronald^{\prime}s\text{ age} \\ y\Rightarrow Megan^{\prime}s\text{ age} \end{gathered}[/tex]

Given that Ronald was 1.5 times older than Megan, we have the equation to be represented

[tex]x=1.5y\text{ ---- equation 1}[/tex]

If Ronald was 27 years old, we have

[tex]x=27[/tex]

Substituting the value of 27 for x into equation 1, we have

[tex]\begin{gathered} 27=1.5y \\ solve\text{ for y by dividing both sides by the coefficient of y,} \\ \frac{27}{1.5}=\frac{1.5y}{1.5} \\ \Rightarrow y=18 \end{gathered}[/tex]

This implies that Megan's age is

[tex]18\text{ years}[/tex]

The textbook isn't helping with the screenshot problem below.

Answers

From the frequency distribution, the measures are given as follows:

a) Total number of observations: 27.

b) The width of each class is of 5.

c) The midpoint of the second class if of 20.5.

d) The modal class is Class 12 - 17.

e) If another class was added, the limits would be 48 - 53.

What is represented by the frequency distribution?

The frequency distribution gives the number of observations that is located in each class.

Hence the total number of observations is given by the sum of the frequencies, as follows:

Total = 9 + 1 + 3 + 8 + 6 = 27.

The modal class is class with the highest number of observations, hence it is of:

Class 12-17.

The width of a class is given by the subtraction of the limits, hence:

Width = 47 - 42 = ... = 23 - 18 = 17 - 12 = 5.

For an added class, the lower bound would be one more than the last class, while the upper bound would be five added to the lower bound, hence the limits are:

48 - 53.

The midpoint of each class is given by the mean of the coordinates, hence, for the second class, it is of:

Midpoint = (18 + 23)/2 = 41/2 = 20.5.

More can be learned about frequency distributions at https://brainly.com/question/24623209

#SPJ1

21. The table below shows the frequency of chirps for the striped ground cricket compared to the ambient temperature, in degrees Fahrenheit.Chirps per SecondTemperature208916722093188417811675157017821569Draw a scatterplot for this data, using x to represent the chirping frequency and y to represent temperature.131415161718192021226768697071727374757677787980818283848586878889909192939495

Answers

Using a graphing calculator :

The scatter plot will look thus:

2 multiplied by the square root of 8

Answers

Answer:[tex]4\sqrt[]{2}[/tex]Explanations:

Note that the square root of 8 is written as:

[tex]\sqrt[]{8}[/tex]

2 multiplied by the square root of 8 will then be expressed as:

[tex]\begin{gathered} 2\sqrt[]{8} \\ 2\text{ }\times\sqrt[]{8} \\ 2\text{ }\times\text{ }\sqrt[]{4}\text{ }\times\text{ }\sqrt[]{2} \\ 2\text{ }\times\text{ 2 }\times\text{ }\sqrt[]{2} \\ 4\text{ }\times\text{ }\sqrt[]{2} \\ 4\sqrt[]{2} \end{gathered}[/tex]

Other Questions
your freezer should be kept at -18 C. One day you woke up and noticed the door was left open and the temperature is now -4 degrees C. How many degrees warmer is the freezer now? help meeeeeeeeeeeeeeeeeeeeeeeeeeeeeeeeeeeeeeee global dispatch corporation and heavy hauling inc. agree in advance to terms that apply to their future e-transactions. this is A can sits on a vertical wooden fencepost 1.9 meters above the ground. Billy picks up a small rock, aims at an angle = 25 above the horizontal and throws the rock, releasing it 1 m above the ground with an initial speed of v0 =10 m/s. Boom! He hits the can! How far away is the fencepost? Which approach to personality emphasizes a desire to achieve higher levels of functioning?. Draw the image of a triangle after a dilation with a scale factor of 2. How many different values can a 16-bit binary number store? How many digits are needed fora hexadecimal number to do the same? How about a base-5 number? Show your work. I took a screenshot I didnt want to type it again I need to balance the equation __kciO3__kcio4+__ KCIAnd How many moles of potassium chloride are produced from 3.90 moles of potassium chlorate ? Answer question number 11. The question is in the image. Statistics and probil Select the sentence from the section "U.S. targets terrorists in the Middle East" that BEST shows HOW the war on terror changed Americans' lives in the United States. Newsela The History of the War on TerrorAThe U.S. led major military attacks in Afghanistan and Iraq in the Middle East.BThey started keeping the prisoners at Guantnamo Bay, which is an American prison in Cuba.CThe National Security Agency, or NSA, began to gather more information about people.DThe country was ruled by the Taliban, an Islamic group. If tan A = 21/20 and cos B = 28/53 and angles A and B are in Quadrant I, find the valueof tan(A - B). Local and analyze Bradford use of allusions to the Bible and conferences to God's intervention in events. What purpose might this device serve in his account Our university consists of three colleges: business, engineering, and fine arts. There are 2,900 students in the business college, 1,500 students in the engineering college, and 1,000 students in the fine arts college. What percent of the total number of students are in the fine arts college. Round your answer to the nearest percent. Find the number of CDs that will produce maximum revenue. How many ounces are in 10 1/2 pounds?1 pound = 16 ounces Factor the following difference of squares. *Check for a GCF. The reaction mixture, at a certain temperature, contained concentrations of0.31 M of NH3, 0.85 M of N2 and 0.031 M of H2 when it reached equilibrium.Calculate Keq at this temperature. Transforming energy is a major trend that is reshaping agribusiness.O TrueO False